Principio de Hamilton modificado que restringe en exceso un sistema al imponer demasiadas condiciones de contorno

En la mecánica hamiltoniana, se muestra una versión del principio de Hamilton para desarrollar un sistema de acuerdo con las mismas ecuaciones de movimiento que el lagrangiano y, por lo tanto, el formalismo newtoniano. En particular, dejar d indicar una variación del camino a través del espacio de fase,

d t 1 t 2 ( pag i q ˙ i H ( pag i , q i , t ) ) d t = 0
se muestra que genera las mismas ecuaciones de movimiento que las encontradas por la transformada de Legendre de las ecuaciones de movimiento de Lagrange. Esencialmente, cuando calculamos las ecuaciones de Euler-Lagrange para el integrando anterior, encontramos
pag ˙ i + H q i = 0 q ˙ i H pag i = 0.
Ahora, dado que ambas son ecuaciones de movimiento de primer orden, requerimos 2 norte condiciones de contorno para obtener una solución, donde norte es el número de partículas. Esto es perfectamente consistente con el formalismo lagrangiano que había 2 norte condiciones de contorno debido a su norte distintas ecuaciones de movimiento de segundo orden.

Una cosa importante a tener en cuenta es que desde pag i q ˙ i H ( pag i , q i , t ) no contiene pag ˙ i dependencia, la variación en pag i ( t ) no necesita ser cero en los puntos finales de la ruta. Este no es el caso con q i ( t ) porque el q ˙ i la dependencia da como resultado la aparición de términos de contorno que deben establecerse en cero para obtener las ecuaciones de movimiento.

Ahora, en lo que se refiere a las condiciones de contorno, esto tiene sentido. Para que el camino se especifique de forma única dada la mecánica newtoniana, requerimos 2 norte condiciones de borde. Estos pueden ser la elección newtoniana habitual de posiciones iniciales y velocidades/momentos, o pueden ser las posiciones inicial y final. Ambos son perfectamente aceptables matemáticamente hablando. Si se requiriera, en la variación del principio de Hamilton modificado, que las variaciones en los momentos también sean cero en los tiempos inicial y final, esto significaría 2 norte condiciones de contorno adicionales, que generalmente restringirían en exceso un sistema newtoniano.

Sin embargo, mi confusión es esta: cuando los libros de texto (Goldstein en particular) consideran funciones generadoras, requieren que el integrando sea invariante a la suma de una derivada de tiempo total de una función F ( q i , pag i , t ) de las coordenadas del espacio de fase. Sin embargo, agregar una función de este tipo en general agregará algunos pag ˙ i dependencia al integrando, que luego agregará términos de contorno a las ecuaciones de movimiento a menos que exijamos que el pag i ( t ) variaciones ser cero en los límites. Esto está bien hasta donde llega. Siempre podemos definir la variación como queramos. El punto es que arroja las ecuaciones de movimiento correctas al final del día. Pero mi confusión radica en cómo esto generalmente no restringe demasiado el sistema. Si especificamos todas las posiciones y momentos tanto en el momento inicial como en el final, ¿no podría ser no newtoniano el camino requerido para conectar esos puntos en el espacio de fase? ¿Importa esto solo si realmente está tratando de usar el principio de acción estacionario para encontrar los caminos, en lugar de solo usarlo para encontrar las ecuaciones de movimiento?

Respuestas (1)

Estas son muy buenas preguntas. referencias 1 y 2 no son del todo consistentes en estos temas.

  1. Analicemos la situación. En general, una versión hamiltoniana del principio de acción estacionaria tiene la forma

    (1) S H [ z ]   =   t i t F d t   L H ( z , z ˙ , t ) ,
    donde el 2 norte -el espacio de fase dimensional tiene coordenadas (no necesariamente canónicas) ( z 1 , , z 2 norte ) . Desde el 2 norte Las ecuaciones EL deben ser ODE de primer orden (a diferencia de las de orden superior) , el integrando
    (2) L H ( z , z ˙ , t )   =   I = 1 2 norte A I ( z , t ) z ˙ I + B ( z , t )
    debe ser una función afín de z ˙ . La variación infinitesimal de la acción hamiltoniana. S H es de la forma
    (3) d S H   =   términos masivos   +   términos-límite ,
    dónde
    (4) términos masivos   =   t i t F d t   I = 1 2 norte d S H d z I d z I
    producir las ecuaciones de Hamilton, y donde
    (5) términos-límite   =   [ I = 1 2 norte L H z ˙ I d z I ] t = t i t = t F   =   0
    debe desaparecer debido a
    (6) norte  condiciones iniciales y  norte  condiciones finales.
    Puesto que hay 2 × 2 norte = 4 norte términos de frontera en la ecuación. (5) pero solo 2 norte condiciones de contorno (BC) (6), no todos los integrandos afines (2) son consistentes. Este desajuste es el núcleo de la pregunta de OP 1 .

    • Algunos de los 4 norte los términos de frontera (5) podrían desaparecer automáticamente si el integrando L H no depende de todas las variables de punto ( z ˙ 1 , , z ˙ 2 norte ) .

    Los términos de contorno restantes (5) deben ser eliminados por los BC (6), que tienen las siguientes posibilidades:

    • Esencial/Dirichlet BC: z I ( t i )   =   z i I y z I ( t F )   =   z F I .

    • CC natural: L H z ˙ I | t i   =   0 y L H z ˙ I | t F   =   0.

    • Combinaciones de los mismos.

    Tenga en cuenta que si los términos restantes son más de 2 norte , entonces algunos de los BC esenciales y naturales deben ser dependientes, es decir, desempeñar un doble papel 2 .

  2. Ahora usemos coordenadas canónicas

    (7) ( z 1 , , z 2 norte )   =   ( q 1 , , q norte ; pag 1 , , pag norte ) .
    referencias 1 y 2 originalmente consideran un lagrangiano hamiltoniano de la forma
    (8) L H   =   j = 1 norte pag j q ˙ j H ( q , pag , t )
    con 2 norte BC esenciales/Dirichlet 3
    (9) q j ( t i )   =   q i j y q j ( t F )   =   q F j ,
    cf. ec. (8.65) en la ref. 1 y ec. (43.8) en la ref. 2. Tenga en cuenta que el hamiltoniano lagrangiano (8) no depende de pag ˙ j . Hacemos hincapié en que los momentos pag j no cumplir BC 3 .

  3. A continuación, consideremos las transformaciones canónicas (CT). Si asumimos que

    (10) j = 1 norte pag j q ˙ j H ( q , pag , t )   =   k = 1 norte PAG k q ˙ k k ( q , PAG , t ) + d F ( q , pag ; q , PAG ; t ) d t
    se mantiene fuera de la cáscara, se sigue a través de manipulaciones algebraicas que
    (11) ecuaciones de Hamilton. y las ecuaciones de Kamilton. son equivalentes.
    referencias 1 y 2 aplican un argumento variacional para deducir (10) (11) por incorrectamente 4 suponiendo un conjunto demasiado completo de 4 norte BC de Dirichlet.

  4. Sin embargo, para CTs de tipos 1-4 es posible dar una prueba variacional de (10) (11) suponiendo únicamente la 2 norte BC (9). En esta publicación Phys.SE relacionada, la prueba para el tipo 1 se proporciona explícitamente.

Referencias:

  1. H. Goldstein, Mecánica Clásica; Secciones 8.5 + 9.1.

  2. LD Landau y EM Lifshitz, Mecánica; § 43 + § 45 .

--

1 Mencionemos que la integral de trayectoria de estado coherente impone 4 norte BC reales, es decir, el sistema está sobrerrestringido. En otras palabras, ¡genéricamente no existen caminos clásicos! Esto está relacionado con la sobrecompletitud de los estados coherentes, cf. por ejemplo, esta publicación de Phys.SE.

2 Curiosamente, este problema no surge para las teorías lagrangianas, donde 4 norte Los BC son el número correcto para 2 norte EDO de segundo orden, cf. por ejemplo , esta publicación Phys.SE relacionada.

3 Después de no imponer correctamente BC en las variables de momento en el texto anterior a la ec. (8.71), ref. 1 da la vuelta en el texto después de la ec. (8.71) y afirma incorrectamente que también se deben imponer BC a las variables de cantidad de movimiento. Esto conduciría a un sistema sobre restringido como OP ya señaló.

4 Véase en el texto entre las ecs. (9.7) y (9.8) en la Ref. 1, y en el texto bajo la ec. (45.5) en la ref. 2.